Use the number line to plot –3, 1, and 3.

Which statements are true? Select all that apply.
–3 > 1
–3 = 3
1 < 3
–3 < 1

Answers

Answer 1

Answer:

Options 3 and 4 are correct

Step-by-step explanation:

-3>1 is false bc a positive is always greater than a negative

-3=3 is false because a positive is not equal to a negative  

1<3 is correct because 1 is less than 3

-3<1 is correct because a negative is always less than a positive.

Answer 2

Answer:

1 < 3

–3 < 1

Step-by-step explanation:


Related Questions

I need help ASAP!!

Can someone explain this? And answer it? I am so confused!!

Answers

Answer:

Step-by-step explanation: hope this helps

Determine the equation of the graph and select the correct answer below.
(1, 1-3)
Courtesy of Texas Instruments

Answers

Answer:

  y = (x -1)² -3

Step-by-step explanation:

A quadratic with a vertex at (h, k) will have an equation of the form ...

 y = a(x -h)² +k

You have (h, k) = (1, -3), and a vertical scale factor* of 1. So, the equation of the graphed curve is ...

  y = (x -1)² -3

_____

* One way to determine the value of "a" in the form shown is to look at the vertical difference between the vertex and the points 1 unit right or left of the vertex. Here, those points are 1 unit above the vertex, so the vertical scale factor "a" is 1.

2.3 repeating as a fraction​

Answers

23/10 = 2 and 3/10




I hope this helped

Answer:

First, we can write:

x = 2 . ¯ 3

Next, we can multiply each side by  10 giving:

10 x = 23 . ¯ 3

Then we can subtract each side of the first equation from each side of the second equation giving:

10 x − x = 23 .¯ 3 − 2 . ¯ 3

We can now solve for  x  as follows:

10 x − 1 x = ( 23 + 0 . ¯ 3 ) − ( 2 + 0 . ¯ 3 )  ( 10 − 1 ) x = 23 + 0 . ¯ 3 − 2 − 0 . ¯ 3  

9 x = ( 23 -2 ) + ( 0 . ¯ 3 − 0 . ¯ 3 )  

9 x = 21 + 0

9 x = 21

9 x /9 = 21 /9

9 x /9 = 3 × 7 /3 × 3

x = 3 × 7 /3 × 3

x = 7 /3

Hope this helps!

Plz mark brainliest! ☜(゚ヮ゚☜)

Find the height of the triangle by applying formulas for the area of a triangle and your knowledge about triangles.
A. 10.5 cm
B. 3.4 cm
C. 8.5 cm
D. 12 cm

Answers

Answer:

12 cm is the right answer pls mark me brainliest

The height of the triangle by applying formulas for the area of a triangle and your knowledge about triangles is 12 cm.

What is Area of Triangle?

The area of a triangle is defined as the total space occupied by the three sides of a triangle in a 2-dimensional plane. The basic formula for the area of a triangle is equal to half the product of its base and height, i.e., A = 1/2 × b × h.

What is Heron's formula?

Heron's formula, formula credited to Heron of Alexandria (c. 62 ce) for finding the area of a triangle in terms of the lengths of its sides. In symbols, if a, b, and c are the lengths of the sides:

Area = √s(s - a)(s - b)(s - c) where s is half the perimeter, or (a + b + c)/2.

Given:

Three sides are: 15cm, 25 cm and 2 cm

Now, Using Heron's formula

semi-perimeter= (25+ 20 + 15)/2

s= 30 cm

Now,

Area of triangle

=√s(s-a)(s-b)(s-c)

=√30* 5 * 10* 15

=√5*2*3*5*2*5*3*5

=5*5*2*3

=150 cm²

Again, area of triangle= 1/2* b* h

150= 1/2* 25* x

12cm= x

Learn more about Area of Triangle here:

https://brainly.com/question/9817285

#SPJ2

1. 3x + 6y = 3 and 7x + 3y = 7
ons for bo​

Answers

Answer:

(1,0)

Step-by-step explanation:

3x + 6y = 3

7x + 3y = 7

Multiply the second equation by -2

-2( 7x + 3y = 7)

-14x -6y = -14

Add this to the first equation to eliminate y

3x + 6y = 3

-14x -6y = -14

--------------------

-11 x = -11

Divide by -11

x = 1

Now find y

3x + 6y = 3

3 +6y = 3

Subtract 3 from each side

6y = 0

y =0

Answer:

x = 1

y = 0

Step-by-step explanation:

3x + 6y = 3

7x  + 3y = 7

=> 3y = 7 - 7x

=> y = -7/3x + 7/3

3x + 6(-7/3x + 7/3) = 3

=> 3x - 14x + 14 = 3

=> -11x = -11

=> -x = -11/11

=> -x = -1

=> x = 1

So, 3(1) + 6y = 3

=> 3 + 6y = 3

=> 6y = 0

=> y = 0/6

=> y = 0

So, x = 1

      y = 0

3. Solve 2log4y - log4 (5y - 12) = 1/2

Answers

Answer:

y =  4  or y = 6

Step-by-step explanation:

2log4y - log4 (5y - 12) = 1/2

​2log_4(y) - log_4(5y-12) = log_4(2)           apply law of logarithms

log_4(y^2) + log_4(1/(5y-12)) = log_4(/2)    apply law of logarithms

log_4(y^2/(5y-12)) = log_4(2)                     remove logarithm

y^2/(5y-12) = 2                                            cross multiply

y^2 = 10y-24                                                  rearrange and factor

y^2 - 10y + 24 = 0

(y-4)(y-6) = 0

y= 4 or y=6

what is the value of a in the functions equation?

Answers

Answer:

y= -2x^2 + 12x -14

Step-by-step explanation:

when I put the points into a graphing calculator this is what I got so I think the answer is -2

Answer:

Step-by-step explanation:

It is -2

Is 7 a solution of 5x - 3 = 12?

Answers

Answer:

no

Step-by-step explanation:

5x - 3 = 12

5x=12+3

x=15/5=3

x=3

substitute x with 7 to find if it is a solution :

5(7)-3=12

35-3 ≠12 ( so no 7 is not a solution)

A researcher performs a hypothesis test to test the claim that for a particular manufacturer, the mean weight of cereal in its 18 ounce boxes is less than 18 ounces. He uses the following hypotheses: H 0: μ = 18 vs H A: μ < 18 and finds a P-value of 0.01. Draw a conclusion about the cereal box weight at a significance level of 0.05.

Answers

Answer:

We conclude that the mean weight of cereal in its 18-ounce boxes is less than 18 ounces.

Step-by-step explanation:

We are given that a researcher performs a hypothesis test to test the claim that for a particular manufacturer, the mean weight of cereal in its 18-ounce boxes is less than 18 ounces.

Let [tex]\mu[/tex] = mean weight of cereal in its 18-ounce boxes.

So, Null Hypothesis, [tex]H_0[/tex] : [tex]\mu[/tex] = 18       {mean that the mean weight of cereal in its 18-ounce boxes is equal to 18 ounces}

Alternate Hypothesis, [tex]H_A[/tex] : [tex]\mu[/tex] < 18      {mean that the mean weight of cereal in its 18-ounce boxes is less than 18 ounces}

Also, it is given that the P-value is 0.01 and the level of significance is 0.05.

The decision rule based on the P-value is given by;

If the P-value of our test statistics is less than the level of significance, then we have sufficient evidence to reject our null hypothesis as our test statistics will fall in the rejection region.If the P-value of our test statistics is more than the level of significance, then we have insufficient evidence to reject our null hypothesis as our test statistics will not fall in the rejection region.

Here, clearly our P-value is less than the level of significance as 0.01 < 0.05, so we have sufficient evidence to reject our null hypothesis as our test statistics will fall in the rejection region.

Therefore, we conclude that the mean weight of cereal in its 18-ounce boxes is less than 18 ounces.

Will Give Brainliest, answer ASAP x=
y=
z=

Answers

Opposite angles are identical.

11y = 55

Y = 55/11

Y = 5

3z-4 = 110

Add 4 to both sides

3z= 114

Z = 114/3

Z = 38

180 -110 = 70

55 + x = 70

X = 70-55

X =15

X = 15, y = 5, z = 38

Answer:x=15, y=5, z=38

Step-by-step explanation: you know that opposite angles are the same. y=5

add 4 to both sides. and you get the value for x y z.

Which of the following is an irrational number?
5 / 4
√5 / 7
1/ 8
3 / 5

Answers

Answer:  [tex]\sqrt{5} /7[/tex]

Step-by-step explanation:

5/4 is not an irrational number because it is already in a fraction the same as 1/8 and 3/5.

The square root of 5 is not rational because it cannot be converted to a fraction or in other words is not a perfect square.

Answer:√5 / 7

Step-by-step explanation:

the perimeter of square is 76 cm find are of square ​

Answers

Answer:

Given the information above, the area of the square is 361 cm²

Step-by-step explanation:

A square is a shape with four equal sides. So, in order to find the area of the square, we must find the length of each individual side. We can do this by dividing the perimeter by 4 because a square has 4 equal sides meaning they have the same lengths.

The perimeter of the square is 76. So, let's divide 76 by 4.

76 ÷ 4 = 19

So, the lengths of each sides in the square is 19cm.

In order to find the area, we must multiply the length and the width together. Since a square has equal sides, then we will multiply 19 by 19 to get the area.

19 × 19 = 361

So, the area of the square is 361 cm²

Answer:

361 cm^2

Step-by-step explanation:

The area of a square can be found by squaring the side length.

[tex]A=s^2[/tex]

A square has four equal sides. The perimeter is the sum of all four sides added together. Therefore, we can find one side length by dividing the perimeter by 4.

[tex]s=\frac{p}{4}[/tex]

The perimeter is 76 centimeters.

[tex]s=\frac{76 cm}{4}[/tex]

Divide 76 by 4.

[tex]s=19 cm[/tex]

The side length is 19 centimeters.

Now we know the side length and can plug it into the area formula.

[tex]A=s^2\\s=19cm[/tex]

[tex]A= (19 cm)^2[/tex]

Evaluate the exponent.

(19cm)^2= 19 cm* 19cm=361 cm^2

[tex]A= 361 cm^2[/tex]

The area of the square is 361 square centimeters.

Solve for x. 3x-91>-87 AND 17x-16>18

Answers

Answer & Step-by-step explanation:

For this problem, we have two inequalities to solve for x.

3x - 91 > -87

17x - 16 > 18

Now that we know what our inequalities are, we will solve them as if we are solving for the value of x.

3x - 91 > -87

Add 91 on both sides.

3x > 4

The solution for the first inequality is 3x > 4

Now let's do the second inequality.

17x - 16 > 18

Add 16 on both sides.

17x > 34

Divide by 17 on both sides.

x > 2

The soultion for the second inequality is x > 2

Answer:

The answer is x>2

Step-by-step explanation:

complete the square to solve
f(x)=x^2-6x+5​

Answers

Answer: See below

Explanation:

x^2 - 6x + 5 = 0
(x^2 - 6x + 9) + 5 - 9 = 0
(x - 3)(x - 3) - 4 = 0
(x - 3)^2 = 4
x - 3 = +-2 (sqrt both sides)

x - 3 = 2
x = 5

x - 3 = -2
x = 1

If x3 + ax2 – bx + 10 is divisible by x2 – 3x + 2,
find the values of
1) a-b
2) 2a-b

Answers

Answer: A=2 and B=13
Explanation: The Factor Theorem states that if a is the root of any polynomial p(x) that is if p(a)=0, then (x−a) is the factor of the polynomial p(x).

Let p(x)=x
3
+ax
2
−bx+10 and g(x)=x
2
−3x+2
Factorise g(x)=x
2
−3x+2:
x
2
−3x+2=x
2
−2x−x+2=x(x−2)−1(x−2)=(x−2)(x−1)
Therefore, g(x)=(x−2)(x−1)
It is given that p(x) is divisible by g(x), therefore, by factor theorem p(2)=0 and p(1)=0. Let us first find p(2) and p(1) as follows:
p(1)=1
3
+(a×1
2
)−(b×1)+10=1+(a×1)−b+10=a−b+11
p(2)=2
3
+(a×2
2
)−(b×2)+10=8+(a×4)−2b+10=4a−2b+18
Now equate p(2)=0 and p(1)=0 as shown below:
a−b+11=0
⇒a−b=−11.......(1)
4a−2b+18=0
⇒2(2a−b+9)=0
⇒2a−b+9=0
⇒2a−b=−9.......(2)
Now subtract equation 1 from equation 2:

(2a−a)+(−b+b)=(−9+11)
⇒a=2
Substitute a=2 in equation 1:
2−b=−11
⇒−b=−11−2
⇒−b=−13
⇒b=13
Hence, a=2 and b=13.

Can someone help me solve parts (a) and (c) please? Thank you!

Answers

a) 4x +6

Add up all the sides to calculate perimeter

Answer:

a) 6x + 6

b) 15 x 24

c) see explanation

Step-by-step explanation:

a) 2x + x + 3 + 2x + x + 3 = 6x + 6

b) 6x + 6 = 78

6x = 72

x = 12

2(12) = 24

(12) + 3 = 15

15 x 24

c) 2x(x + 3) = 2x² + 6x

2(12)² + 6(12) = 288 + 72 = 360

15 x 24 is also 360

Pls help, I don’t know how to fo

Answers

frustum of a cone is: = pi * l(R + r)

(l) = slant height of the frustum.

from 2929.645714 - 506.1257143

= 2423.52

= 2423.5cm

Answer:

from 2929.645714 - 506.1257143

= 2423.52

= 2423.5cm

What is the least common multiple of all positive integers smaller than 8?

Answers

Answer:

420

Step-by-step explanation:

We need to find the LCM of 1, 2, 3, 4, 5, 6 and 7. The LCM of 1, 2, and 4 is 4 and the LCM of 3 and 6 is 6 so the list becomes the LCM of 4, 5, 6 and 7. The LCM of 4 and 5 is 20, the LCM of 20 and 6 is 60 and the LCM of 60 and 7 is 420.

Answer:

420

Step-by-step explanation:

Prime factorization:

7 = 7

6 = 2 × 3

5 = 5

4 = 2 × 2

3 = 3

2 = 2

1 =  1

LCM: 7, 6, 5, 4, 3, 2, 1

2 × 2 × 3 × 5 × 7  = 420

Georgina has $5.60 in quarters and dimes. If Georgina has 14 more dimes than quarters, how many quarters does she have?
[I don't want the answer i just want someone to show me how the problem is set up because its confusing me...Thanks]

Answers

Answer:

She has 12 quarters.

Step-by-step explanation:

You have two unknowns, the number of quarters and the number of dimes. You need one equation per unknown, so you need two equations. One equation deals with the number of coins. The other equation deals with the values of the coins.

The first step is to choose variables. We can go with x and y, but I prefer to choose q and d, so I remember more easily what they represent.

Let q = number of quarters.

Let d = number of dimes.

Let's deal with the numbers of coins first.

"Georgina has 14 more dimes than quarters"

She has q number of quarters, but the number of dimes is 14 more than the number of quarters. d is 14 more than q. Our first equation is

d = q + 14

Now we deal with the values of the coins.

We don't know yet the actual numbers of dimes and quarters, so we use d and q to represent those numbers.

One dime is worth $0.10; d number of dimes is worth 0.1d.

A quarter is worth $0.25; q number of quarters is worth 0.25q.

The total value of the dimes and quarters is the sum of the values of the two sets of coins:

0.1d + 0.25q

We are told "Georgina has $5.60 in quarters and dimes."

This gives us our second equation.

0.1d + 0.25q = 5.6

Now we have a set of two equations in two variables:

d = q + 14

0.1d + 0.25q = 5.6

There are several methods for solving a system of equations. Since the first equation is already solved for a variable, d, we can use the substitution method.

We rewrite the second equation, but we substitute q + 14 for d in the second equation.

Second equation:

0.1d + 0.25q = 5.6

Substitute q + 14 for d:

0.1(q + 14) + 0.25q = 5.6

Distribute on the left side:

0.1q + 1.4 + 0.25q = 5.6

Combine like terms on the left side:

0.35q + 1.4 = 5.6

Subtract 1.4 from both sides:

0.35q = 4.2

Divide both sides by 0.35:

q = 12

There are 12 quarters.

(You are asked only the number of quarters, so you can stop here. I will continue to find also the number of dimes for 2 reasons. 1) You see how it's done, so it will help with other problems. 2) By finding the numbers of dimes and quarters, then we can check if our solution is correct, which I will do below at the end.)

Now we use the first equation, d = q + 14. We substitute 12 for q and solve for d.

d = q + 14

d = 12 + 14

d = 26

There are 26 dimes.

Check:

We check the numbers of coins:

26 - 12 = 14 The number of dimes is indeed 14 more than the number of quarters.

We check the values of the coins:

0.1(26) + 0.25(12) = 2.6 + 3 = 5.6 The value of the coins is indeed $5.60.

Our answer is correct.

If the measure of angle 4 is (11 x) degrees and angle 3 is (4 x) degrees, what is the measure of angle 3 in degrees?

Answers

Answer:

is it 2

Step-by-step explanation:

In which direction must the graph of Ax) = x be shifted to produce the graph of g(x) = f(x) - 4?
ОА. up
OB. down
O c. left and down
OD. right and up​

Answers

Answer: B. down

Step-by-step explanation:

Translation rules:

For a function h(x):

h(x+c) is a left-shift by c units.h(x-c) is a right-shift by c units.h(x)+c is a up-shift by c units.h(x)-c is a down-shift by c units.

Here, the graph of f(x) becomes the graph of g(x) =f(x)-4 which is similar to "h(x)-c".

That means , f(x) is shifted 4 units down to become g(x).

So, correct option : B. down

Help the question is there

Answers

Answer:

y = 7 when x = -5

Step-by-step explanation:

First go to x = -5

Then go up to where you meet the green line

The y value is 7

y = 7 when x = -5

Latanya buys 5 yard of blue fabric and 8 yards of green fabric. the blue fabric cost $2 dollars more than the green fabric.she pays a total of $ 62. what would be the combined cost of 1 yard of blue fabric and one yard of green fabric?

Answers

Answer: $10

Step-by-step explanation:

let x = the price of green fabric, then x+2 = blue fabric price

8x+5(x+2)=62

8x+5x+10=62

    13x+10=62

          13x=52

              x=4

price of green fabric=$4

price of blue fabric=$6

4+6=$10

PLEASE HELP!!

Complete the equation.

(13)^2 (13)^-14 (13)^5 = 13 [ ]a0

Answers

Answer:

(13)^2 (13)^-14 (13)^5

=2197

Answer:

13^3

Step-by-step explanation:

-4+2+5=3

is it . that or something else ??

If you make $3.80 an hour plus tips, what is your paycheck for the week if you worked 40 hours and made $250.00 dollars in tips?

Answers

Answer:

$402

Step-by-step explanation:

Hello!

If you made 3.80 an hour and worked 40 we can multiply these to find the total amount you earned.

3.80 * 40 = 152

You also made 250 in tips so we add that to the total

152+250 = 402

The answer is $402

Hope this helps!

I would make $402.00 by the end of the week

Please help me answer this question. -15 - g/3 = -5.
What is g?​

Answers

Answer:

g = -30

Step-by-step explanation:

-15 - g/3 = -5

Add 15 to each side

-15+15 - g/3 = -5+15

-g/3=10

Multiply by -3 to each side

-g/3 * -3  = 10*-3

g = -30

Answer:

g= -30

Step-by-step explanation:

after writing down the problem, multiply both sides of the equation by 3 to get  -45-g= =15. then subtract 15 from both sides to get g=-30, which is your answer. hope this helps!

Which of the following is true? Tangent is positive in Quadrant I. Sine is negative in Quadrant II. Cosine is positive in Quadrant III. Sine is positive in Quadrant IV.

Answers

A) Tangent is positive in Quadrant I.

Since sine and cosine are both positive in Quadrant I and tangent is the ratio of sine to cosine, tangent is positive in Quadrant I

Answer:

A

Step-by-step explanation:

I had this question and got it right the user above explains it in detail

Please answer quickly! A radio telescope has a parabolic surface, as shown below. A parabola opening up with vertex at the origin is graphed on the coordinate plane. The height of the parabola from top to bottom is 1 meter and its width from left to right is 20 meters. If the telescope is 1 m deep and 20 m wide, how far is the focus from the vertex?

Answers

Answer:

Basing on the description, a parabola checking with vertex at origin, the formula with vertex at origin can be used, x^2 = 4py. p is the focus therefore with the dimensions given, we get yourself a 0.25 and this is the distance of the focus to the vertex.

A blimp is 1100 meters high in the air and measures the angles of depression to two stadiums to the west of the blimp. If those measurements are 75.2° and 17.9°, how far apart are the two stadiums?

Answers

Answer:

The two stadiums are approximately 3115.1 meters away from each other

Step-by-step explanation:

Since we can construct two right angle triangles between the blimp and the two stadiums as shown in the attached image, then the distance "x" between the two can be find as the difference between the right triangle legs that extend on the ground.

In order to find the size of such legs, one can use the tangent function of the given depression angles as shown below:

[tex]tan(75.2^o)=\frac{1100}{a} \\a=\frac{1100}{tan(75.2^o)}\\a\approx 290.6\,\,meters[/tex]

and for the other one:

[tex]tan(17.9^o)=\frac{1100}{b} \\b=\frac{1100}{tan(17.9^o)}\\b\approx 3405.7\,\,meters[/tex]

The the distance between the stadiums is the difference:

b - a = 3405.7  - 290.6 meters = 3115.1  meters

need help will give you a good rating us

Answers

Answer:

[tex]\boxed{2\pm \frac{\sqrt{2}}{2}}[/tex]

Step-by-step explanation:

[tex]2x^2-8x=-7[/tex]

[tex]\sf Add \ 7 \ on \ both \ sides}.[/tex]

[tex]2x^2-8x+7=-7+7[/tex]

[tex]2x^2-8x+7=0[/tex]

[tex]ax^2 +bx+c=0[/tex]

[tex]\sf Apply \ quadratic \ formula.[/tex]

[tex]a=2 \ \ \ b=-8 \ \ \ c = 7[/tex]

[tex]x=\frac{-b\pm\sqrt{b^2-4ac}}{2a}[/tex]

[tex]x=\frac{-(-8)\pm\sqrt{(-8)^2-4(2)(7)}}{2(2)}[/tex]

[tex]x=\frac{8\pm\sqrt{64-56}}{4}[/tex]

[tex]x=\frac{8\pm\sqrt{8}}{4}[/tex]

[tex]x=\frac{8\pm2\sqrt{2}}{4}[/tex]

[tex]x=2\pm \frac{\sqrt{2}}{2}[/tex]

Other Questions
"To Kill a Mockingbird" is a literary representation of people dealing with all sorts of judgments and differences. Aptly, at one point Jem describes four kinds of people in Maycomb County: "Our kind of folks don't like the Cunninghams, the Cunninghams don't like the Ewells, and the Ewells hate and despise the colored folks." Is "otherness" rooted in people? How does our society deal with those differences today? When she graduates college, Linda will owe $43,000 in student loans. The interest rate on the federal loans is 4.5% and the rate on the private bank loans is 2%. The total interest she owes for one year was $1,585. What is the amount of each loan? The economy in the Southern colonies was based on What do nitrifying bacteria do? Company's budgeted prices for direct materials, direct manufacturing labor, and direct marketing (distribution) labor per attach case are $39, $7, and $12, respectively. The president is pleased with the following performance report: Actual Costs Static Budget VarianceDirect materials 564,000 $400,000 $36,000 FDirect manufacturing labor 78,000 80 2,000 FDirect marketing (distribution) labor 110,000 120,000 10,000FActual output was 9,100 attach cases. Assume all three direct-cost items above are variable costs. Requirement: a. Is the president's pleasure justified? b. Prepare a revised performance report that uses a flexible budget and a static budget. 5) What are the states annexed by Doctrine of lapse? find the multiplicative inverse of 3 by 4 minus 5 by 7 George is designing ledges for the octagonal ( 8 side ) gazebo. All sides are equal length, and each ledge much be 18 inches shorter then the sides. a: what is the minimum length of wood he should purchase if the perimeter of the gazebo is 64 feet ?b : what as your solution process Corinne was shaken after learning that a _[blank] occurred during the auto accident.Which word form correctly completes the sentence?O fatalityO fatalisticO fatalO fatally Find the sum of (5x3 + 3x2 - 5x + 4) and (8x3 -5x2 + 8x + 9) Water flows from a bathroom tap at a rate of 2 gallons every 6 seconds. At this rate, how many minutes will it take to fill an 80-gallon tub? the distance between two successive troughs of wave is 0.4m. If the frequency of the source is 825Hz, calculate the speed of the wave help...help...asap... A study collects samples of water from the tap in Vacaville and from bottled water available from the Nugget stores and samples their pH levels. The results are in the table below. I find a bottle marked #13 but cannot read the label for the type of water. He measures the pH and gets 6.32. What type of water do you think it is? hat a 15 kg body is pulled along a horizontal fictional table by a force of 4N what is the acceleration of the body The pepper plant has 2/3 as many fruits on it as the tomato plant has. The tomato plant has 9 fruits on it. How many fruits does the pepper plant have on it? What is 5 feet and 11 inches in inches Two students are comparing scientific experiments to investigations. They came up with the following ideas. Student A: Testing plant growth in different types of soils Student B: Comparing the water level in a lake during different times of the year Which student gave an example of a scientific experiment? Student A because it requires experimental and control groups Student B because it requires a hypothesis Student A because it is preferably done in natural settings Student B because it is preferably done across a long period of time a company should stop making a part internally and buy externally when 14. Twice the sum of a number and eight